Haag's Theorem: Importance & Implications in QFT

In summary, Haag's theorem states that the S operator in QFT, initially assumed to be unitary, is not unitary anymore due to its interaction with fields. This is due to the fact that the Interaction Picture does not exist in QFT. The importance of this theorem lies in the fact that it applies to rigorous QFT where the aim is to construct a Lorentz invariant QFT that exists for all energies. However, in 3+1 dimensions, most QFTs are believed not to exist at all, unless they are asymptotically free or safe, making it difficult to construct a non-asymptotically free theory. This theorem also has implications for defining the Hamiltonian as a self-adjoint operator
  • #1
Qubix
82
1
So I'm currently studying QFT, and I got to the point where I realized that the S operator, initially assumed to be unitary, is not unitary anymore, since it is assumed to act between t0 = - infinity and tf = infinity. The author of the book I'm using says this is due to Haag's Theorem, so I gave that a search, and found a rather interesting bunch on information.

http://en.wikipedia.org/wiki/Haag's_theorem

It seems that the Interaction Picture does not exist in QFT. What is the importance of this theorem and what are its other implications?
 
Physics news on Phys.org
  • #2
Avodyne said:
See also section 10.5, "How to stop worrying about Haag's theorem", in The Conceptual Framework of Quantum Field Theory by Anthony Duncan (Oxford, 2012).

Also, the interaction Hamiltonian doesn't exist as a genuine mathematical object. From Folland's "Quantum Field Theory for Mathematicians: A Tourist's Guide for Mathematicians" (a book that I highly recommend) page 123

"... the ##H_I##'s that we shall need are too singular."

Too singular, because they products of distributions. From page 180

"A precise mathematical construction of the interacting fields that describe actual fundamental physical processes in 4-dimensional space-time is still lacking and may not be feasible without serious modifications to the theory. Similarly, we have no way to define the Hamiltonian ##H## in a mathematically rigorous way as a self-adjoint operator. It was presented as the sum of the free Hamiltonian ##H_0##, which is well-defined and the interaction Hamiltonian ##H_I##; but the latter was presented as the integral of a density consisting of products of fields, which are operator-valued distributions rather than functions."
 
  • #3
Most physical QFT is not rigourous. You can roughly think of it as a many-body system on a lattice and QFT emerges at low energies. This sort of thinking works for QCD, but there is some problem for chiral gauge theories. Regardless the idea is that physical QFTs are only low energy effective theories, and may not exist at all energies. http://quantumfrontiers.com/2013/06/18/we-are-all-wilsonians-now/

In rigourous QFT, the aim is to construct a Lorentz invariant QFT that exists for all energies. Haag's theorem applies to such theories. I've found these comments about the theorem useful.

http://www.rivasseau.com/resources/book.pdf, p15: "The Gell-Mann Low formula ... is difficult to justify because the usual argument, based on the so called "interaction picture" is wrong, by a theorem of Haag ... Euclidean formulation ... theorems which allow us to go back from Euclidean to Minkowski space ..."

Also, https://webspace.utexas.edu/lupher/www/papers/KronzLupherPreprint.pdf , p7: "If representations are unitarily inequivalent, one is left to wonder whether they are at least empirically equivalent in some sense. If a reasonable notion of empirical equivalence cannot be found, then it will be necessary to introduce criteria for representation selection. To resolve this selection problem, Haag and Kastler (1964) introduced the notion of physical equivalence, which is related to Fell’s (1960) notion of weak equivalence, and a theorem proved by Fell, which they reformulate in terms of physical equivalence."
 
Last edited by a moderator:
  • #5
atyy said:
In rigourous QFT, the aim is to construct a Lorentz invariant QFT that exists for all energies. Haag's theorem applies to such theories.
In 3+1 dimensions, most QFTs are believed not to exist at all (only asymptotically free theories are believed to exist). So any attempt at a rigorous construction of a non-asymptotically free theory in 3+1 dimensions should fail.

See also section 10.5, "How to stop worrying about Haag's theorem", in The Conceptual Framework of Quantum Field Theory by Anthony Duncan (Oxford, 2012).

Short version: QFT requires an ultraviolet regulator (such as a lattice), and Haag's theorem does not apply when the regulator is in place.
 
  • #6
Avodyne said:
In 3+1 dimensions, most QFTs are believed not to exist at all (only asymptotically free theories are believed to exist). So any attempt at a rigorous construction of a non-asymptotically free theory in 3+1 dimensions should fail.

See also section 10.5, "How to stop worrying about Haag's theorem", in The Conceptual Framework of Quantum Field Theory by Anthony Duncan (Oxford, 2012).

Short version: QFT requires an ultraviolet regulator (such as a lattice), and Haag's theorem does not apply when the regulator is in place.

3+1? You must be a squalid state physicist who can't add. All HEP people say 4 :)

(I'm a biologist.)
 
  • #7
For people seeing physics beyond the Standard Model, then 3+1 instead of 4 is a must. :smile:
 
  • #8
Stating the signature 3 + 1 is essential.

dextercioby said:
For people seeing physics beyond the Standard Model, then 3+1 instead of 4 is a must. :smile:

An for people working with models in 1 + 1 and 2 + 1.
 
  • #9
Avodyne said:
In 3+1 dimensions, most QFTs are believed not to exist at all (only asymptotically free theories are believed to exist). So any attempt at a rigorous construction of a non-asymptotically free theory in 3+1 dimensions should fail.

See also section 10.5, "How to stop worrying about Haag's theorem", in The Conceptual Framework of Quantum Field Theory by Anthony Duncan (Oxford, 2012).

Short version: QFT requires an ultraviolet regulator (such as a lattice), and Haag's theorem does not apply when the regulator is in place.

First of all, what is an "asymptotically free theory" ? Second, I don't know how you apply a lattice to QFT. From what wikipedia says, "Every lattice in [tex]ℝ^n[/tex] can be generated from a basis for the vector space by forming all linear combinations with integer coefficients." So taking linear combinations of the eigenbasis of a Hilbert space, with integer coefficients gives you a lattice?
 
  • #11
One formal possibility in addition to asymptotic freedom is asymptotic safety. However, as Avodyne says, currently no 3+1 theory has been shown to be asymptotically safe without being asymptotically free. It is being researched if 3+1 gravity might be asymptotically safe, even though it is not asymptotically free.

Asymptotic freedom/safety are physics concepts and not rigourous. However, it is widely believed that it is not worth attempting a rigourous construction of a theory, unless it has been shown to be asymptotically free or safe. 3+1d Yang-Mills is asymptotically free, which is why its rigourous construction has been posed as a Clay Millenium problem.
 
Last edited:
  • #12
Avodyne said:
See also section 10.5, "How to stop worrying about Haag's theorem", in The Conceptual Framework of Quantum Field Theory by Anthony Duncan (Oxford, 2012).

Short version: QFT requires an ultraviolet regulator (such as a lattice), and Haag's theorem does not apply when the regulator is in place.
That's exactly what I wanted to say.
 
  • #13
Could this lattice be associated with a fundamental discreteness of spacetime in the Planck regime or are these different concepts?
 
  • #14
We have to distinguish between the real world and mathematical formulations of a particular theory. We've been discussing the latter here. In this context, the lattice is just a technical tool.

True discreteness at the Planck scale is an intriguing possibility. That would provide a real-world ultraviolet cutoff for the Standard Model. But then there are many other possibilities for an "ultraviolet completion" of the Standard Model + gravity that don't involve fundamental discreteness (e.g. zillions of string models).
 
  • Like
Likes 1 person
  • #15
Avodyne said:
Short version: QFT requires an ultraviolet regulator (such as a lattice), and Haag's theorem does not apply when the regulator is in place.
QFTs do not require a regulator, although perhaps you mean this in a practical sense, i.e. to work with QFTs at our current level of mathematics requires a regulator at intermediate steps.

Also Haag's theorem is not affected by the ultraviolet cutoff, Haag's theorem applies even with an ultraviolet cutoff in place. You need an infrared cutoff to prevent Haag's theorem.
 
  • #16
DarMM said:
Also Haag's theorem is not affected by the ultraviolet cutoff, Haag's theorem applies even with an ultraviolet cutoff in place. You need an infrared cutoff to prevent Haag's theorem.
To prevent Haag's theorem, is the IR cutoff enough, or do we need both UV and IR cutoff?
 
  • #17
DarMM said:
QFTs do not require a regulator, although perhaps you mean this in a practical sense, i.e. to work with QFTs at our current level of mathematics requires a regulator at intermediate steps.
Can you be more specific how, at least in principle, one can work with QFT without a regulator?
 
  • #18
BPHZ approach does not use a regulator.
 
  • #19
andrien said:
BPHZ approach does not use a regulator.
OK, but BPHZ method subtracts an infinite quantity from an infinite quantity, which is not well defined at the mathematical level. The Haag's theorem, on the other hand, assumes that one works with quantities which are well defined mathematically. Physically, the BPHZ method postulates that the mathematically ill defined expression is actually equal to a finite coupling constant extracted from experiments. In this way, BPHZ is also a dirty trick which replaces an infinite quantity by a finite one, so in this sense it is not very different from a "regularization".
 
Last edited:
  • #20
Demystifier said:
To prevent Haag's theorem, is the IR cutoff enough, or do we need both UV and IR cutoff?

Demystifier said:
Can you be more specific how, at least in principle, one can work with QFT without a regulator?

As I understand it, physical QFT does not require it to exist at all energies, a cut-off is fine, and QFT is just a low energy effective theory.

I think DarMM is saying that there are nonetheless some nonlinear relativistic QFTs that exist at all energies, and in infinite volume. In my understanding, the AdS/CFT proposal assumes that the CFT exists for all energies, which is why it is conjectured to provide a non-perturbative definition of some sector of string theory.

http://www.claymath.org/sites/default/files/yangmills.pdf (p8) mentions that relativistic QFTs that exist at all energies, and in infinite volume, have been constructed in fewer than 3+1 spacetime dimensions.
 
  • #21
atyy said:
Demystifier said:
Can you be more specific how, at least in principle, one can work with QFT without a regulator?
As I understand it, physical QFT does not require it to exist at all energies, a cut-off is fine, and QFT is just a low energy effective theory.

A cut-off is (one of the types of) regularization.
 
  • #22
George Jones said:
A cut-off is (one of the types of) regularization.

Yes, physically-relevant QFT, like the standard model, requires a cut-off. However, the article I linked to goes on to say that nonlinear relativistic QFTs in fewer than 4 spacetime dimensions in infinite volume at all energies have been rigourously constructed. Yang-Mills in 4D is asymptotically free, which is why it is a candidate for rigourous construction of a 4D QFT without a cut-off.
 
Last edited:
  • #23
DarMM said:
QFTs do not require a regulator
They don't require a regulator if they exist. But it is widely believed by experts that, in 3+1D, only asymptotically-free theories exist.

DarMM said:
Also Haag's theorem is not affected by the ultraviolet cutoff, Haag's theorem applies even with an ultraviolet cutoff in place.
I disagree, and my opinion is backed up by Duncan's book.
 
  • #24
Avodyne said:
I disagree, and my opinion is backed up by Duncan's book.

Duncan seems to say on his p369 that both the ultraviolet cutoff and the finite volume cut-off are needed to have the interaction picture. IIRC, sometime ago on these forums Dr Du said the same thing about condensed matter in the infinite volume limit. In condensed matter physically it can't really matter, since physically any material in the lab has a natural ultraviolet cutoff and finite volume.
 
  • #25
I disagree, and my opinion is backed up by Duncan's book.
Haag's theorem is a theorem proving the non-existence of the interaction picture provided the theory is translation invariant, i.e. if there is no infrared cutoff, it does not discuss the ultraviolet cutoff.

Basically Haag's theorem proves that free and interacting theories are unitarily inequivalent unless an infrared cutoff is in place.

There is no such general result for ultraviolet cutoffs because there are counterexamples. For example ##\phi^4## without an ultraviolet cutoff (but with an infrared cutoff) is unitarily equivalent to the free theory in 1+1 dimensions and so the interaction picture does exist.

For interaction picture to exist sometimes both an ultraviolet and infrared cutoff is needed, sometimes only an infrared cutoff. Haag's theorem is the statement that an infrared cutoff is always needed for the interaction picture to exist.
 
  • Like
Likes Demystifier
  • #26
Demystifier said:
To prevent Haag's theorem, is the IR cutoff enough, or do we need both UV and IR cutoff?
As above, Haag's theorem is the statement that you always need an IR cutoff. Sometimes, although it is rare, the interaction picture can exist without an ultraviolet cutoff.
 
  • Like
Likes Demystifier
  • #27
Demystifier said:
Can you be more specific how, at least in principle, one can work with QFT without a regulator?
A QFT doesn't require a regulator, in the sense that it can exist mathematically without one. For practical calculations however one almost always needs a cutoff, since we currently aren't capable of directly solving a QFT without help from a field theory, which is singular with respect to the interacting theory and so we see infinities.

However there is the Epstein-Glaser formalism which requires no cutoffs (Infrared or ultraviolet) nor subtracting infinite quantities, instead it uses the Hahn-Banach theorem.
 
  • #28
Avodyne said:
They don't require a regulator if they exist. But it is widely believed by experts that, in 3+1D, only asymptotically-free theories exist.
Yes true, so quantum field theories in general do not require a regulator. Only the ones without a continuum limit do.
 
  • #29
Examples for IR cutoffs are torus compactifications (which do not break translational invariance but require e.g. periodic boundary conditions)

Examples for theories w/o UV cutoffs are super-renormalizable theories in 1+1 dimension.
 
  • #30
DarMM said:
For interaction picture to exist sometimes both an ultraviolet and infrared cutoff is needed, sometimes only an infrared cutoff. Haag's theorem is the statement that an infrared cutoff is always needed for the interaction picture to exist.

DarMM said:
As above, Haag's theorem is the statement that you always need an IR cutoff. Sometimes, although it is rare, the interaction picture can exist without an ultraviolet cutoff.
Thank you for the clarification.
 
  • #31
DarMM said:
However there is the Epstein-Glaser formalism which requires no cutoffs (Infrared or ultraviolet) nor subtracting infinite quantities, instead it uses the Hahn-Banach theorem.
Where can I learn more about that?
 
  • #32
DarMM said:
since we currently aren't capable of directly solving a QFT without help from a field theory
Sorry this should read "without help from a free field theory". The exception being lattice formulations, which solve theories without reference to the free theory. However in that case we need a cutoff since we have no mathematical control over the continuum path integral.
 
  • #33
DarMM said:
Haag's theorem is a theorem proving the non-existence of the interaction picture provided the theory is translation invariant, i.e. if there is no infrared cutoff, it does not discuss the ultraviolet cutoff.

Basically Haag's theorem proves that free and interacting theories are unitarily inequivalent unless an infrared cutoff is in place.

There is no such general result for ultraviolet cutoffs because there are counterexamples. For example ##\phi^4## without an ultraviolet cutoff (but with an infrared cutoff) is unitarily equivalent to the free theory in 1+1 dimensions and so the interaction picture does exist.

For interaction picture to exist sometimes both an ultraviolet and infrared cutoff is needed, sometimes only an infrared cutoff. Haag's theorem is the statement that an infrared cutoff is always needed for the interaction picture to exist.

Ok, so what is Infrared cutoff / UV cutoff, specifically, what is a cutoff ? An upper/lower bound placed on the energy / frequency spectra? If so, by an infinite volume cutoff, do you mean the volume that appears in all field equation solutions of QFT (Klein-Gordon, Dirac, Vector) is imposed as a finite (even though large) volume ?
 
  • #34
DarMM said:
A QFT doesn't require a regulator, in the sense that it can exist mathematically without one. For practical calculations however one almost always needs a cutoff, since we currently aren't capable of directly solving a QFT without help from a field theory, which is singular with respect to the interacting theory and so we see infinities.

However there is the Epstein-Glaser formalism which requires no cutoffs (Infrared or ultraviolet) nor subtracting infinite quantities, instead it uses the Hahn-Banach theorem.

http://www.rivasseau.com/resources/book.pdf (p15) mentions some of Epstein and Glaser's work, but it seems that it only provides "rigourous justification" in the sense of "formal power series". Is that enough to define a theory?
 
  • #35
DarMM said:
However there is the Epstein-Glaser formalism which requires no cutoffs (Infrared or ultraviolet) nor subtracting infinite quantities, instead it uses the Hahn-Banach theorem.

Demystifier said:
Where can I learn more about that?
In the meantime, I have learned something about Epstein-Glaser approach from the book:
G. Scharf, Finite Quantum Electrodynamics: The Causal Approach
which is a book completely devoted to the Epstein-Glaser theory and its developments.

In particular, at the top of page 180 the author writes:
"This is an ultraviolet "regularization" in the usual terminology. It should be stressed, however, that here this is a consequence of the causal distribution splitting and not an ad hoc recipe".

Therefore, I do not think it is correct to say that there is no regularization in the Epstein-Glaser approach. It's only that the regularization is mathematically better justified.

More technically, the Epstein-Glaser approach starts from the observation that time ordering of field operators introduces step functions theta(t-t'), which are ill defined at zero. Therefore, the theta functions are replaced by certain better defined regularized functions, which avoid problematic UV divergences.

For a very brief review of Epstein-Glaser approach see also
http://www.google.hr/url?sa=t&rct=j...pwW1s-CYzKoSs4Wmu3o8i1Q&bvm=bv.58187178,d.bGQ

By the way, Glaser was a Croat
http://en.wikipedia.org/wiki/Vladimir_Glaser
just as I am. :smile:
 
Last edited:

Similar threads

Replies
5
Views
1K
  • Quantum Physics
Replies
24
Views
8K
  • Quantum Physics
Replies
1
Views
2K
Replies
39
Views
12K
  • Quantum Physics
Replies
9
Views
6K
  • Quantum Physics
3
Replies
79
Views
17K
  • Programming and Computer Science
Replies
29
Views
3K
Replies
36
Views
7K
  • Electromagnetism
Replies
16
Views
2K
  • Quantum Physics
2
Replies
68
Views
9K
Back
Top